Integration Problem Solution Check: Is it Correct?

  • Thread starter Thread starter mdnazmulh
  • Start date Start date
  • Tags Tags
    Integration
Click For Summary
The integration problem presented involves the integral of (1 - x/a)^(1/n) multiplied by x. The initial solution was deemed overly complicated, with suggestions to simplify the approach. Instead of using integration by parts, it was recommended to directly multiply and integrate the expression az^(1/n) - z^((n+1)/n). This method is expected to yield a more straightforward solution. The discussion emphasizes the importance of simplifying integration techniques for better accuracy and efficiency.
mdnazmulh
Messages
51
Reaction score
0

Homework Statement



I solved the integration problem but i am not sure whether my solution is correct or wrong. I attached the picture of the solution.. Kindly tell me whether it is correct or wrong.

Homework Equations



This was the problem :
\int(1- \frac{x}{a} )^{\frac{1}{n}} x dx

The Attempt at a Solution


 

Attachments

  • sltion.jpg
    sltion.jpg
    46.2 KB · Views: 404
Physics news on Phys.org
I didn't check the whole thing since your solution is far more complicated than necessary. After you have, correctly, \int z^{1/n}(a- z)dz, do NOT use "integration by parts", just multiply:
\int (az^{1/n}- z^{1+1/n})dz= \int (az^{1/n}- z^{(n+1)/n})dz
 
Question: A clock's minute hand has length 4 and its hour hand has length 3. What is the distance between the tips at the moment when it is increasing most rapidly?(Putnam Exam Question) Answer: Making assumption that both the hands moves at constant angular velocities, the answer is ## \sqrt{7} .## But don't you think this assumption is somewhat doubtful and wrong?

Similar threads

  • · Replies 105 ·
4
Replies
105
Views
6K
Replies
3
Views
2K
Replies
7
Views
2K
  • · Replies 19 ·
Replies
19
Views
2K
Replies
7
Views
2K
Replies
9
Views
1K
  • · Replies 9 ·
Replies
9
Views
2K
  • · Replies 22 ·
Replies
22
Views
3K
Replies
4
Views
3K
  • · Replies 9 ·
Replies
9
Views
1K